gmatalongthewatchtower
Thanks Received: 1
Jackie Chiles
Jackie Chiles
 
Posts: 47
Joined: November 22nd, 2011
 
 
 

Q13 - Editorialist : Some people argue that ramps and other

by gmatalongthewatchtower Fri Jul 20, 2012 9:25 pm

The question says "If ramps are installed, wheelchair guys will visit businesses"

The correct answer to "most strongly support" says
"If no ramps, no wheelchair guys" How could this be true? Thoughts?


Thanks
User avatar
 
ManhattanPrepLSAT1
Thanks Received: 1909
Atticus Finch
Atticus Finch
 
Posts: 2851
Joined: October 07th, 2009
 
 
 

Re: Q13 - Editorialist : Some people argue that ramps and other

by ManhattanPrepLSAT1 Mon Jul 23, 2012 2:55 pm

gmatalongthewatchtower Wrote:The question says "If ramps are installed, wheelchair guys will visit businesses"

The correct answer to "most strongly support" says
"If no ramps, no wheelchair guys" How could this be true? Thoughts?

The issue is that this is not what answer choice (E) is saying. The correct answer (E) is an explanation for why places that had no ramps or wheelchair access and had very little use by wheelchair users suddenly are visited by people in wheelchairs once ramps and wheelchair access is installed.

Answer choice (E) says that the reason why some areas are not frequented by wheelchair users is simply because they are not accessible to wheelchair users.

Incorrect Answers

(A) may be a reason why no wheelchair access is available in many areas but is not supported by the influx of wheelchair users once the wheelchair access is installed.
(B) is not supported since the statements do not address profits.
(C) is not supported since the statements do not address profits.
(D) is too strong. We have no information to support the claim that "most" businesses are not wheelchair accessible.
 
robinzhang7
Thanks Received: 1
Vinny Gambini
Vinny Gambini
 
Posts: 20
Joined: January 28th, 2015
 
 
 

Re: Q13 - Editorialist : Some people argue that ramps and other

by robinzhang7 Wed Jul 22, 2015 5:13 pm

Hi,

I was confused why the answer isn't (A). (A) seems to just be a restatement of the editorialist's final statement:

The stimulus follows as such:

Some people hold this view: Areas not frequented by WU --> installations unnecessary

Editorialist's view: Installations made --> Frequented by WU

It seems to me the stimulus relies on the same conditional statement (one just the contrapositive of the other) which is in line with answer choice (A).

However (E) is basically saying this: NO installations <--> (some) NOT frequented by WU
(E) works as well because formal logic allows the conditionals in the stimulus to turn into "some" statements. However, if answer choice (A) is basically the SAME as the conditionals in the stimulus, why is not (A) the answer as well!??

The only reason I can think of to eliminate (A) is that this answer choice talks about "reluctance" - something we have no idea about. If the answer choice had said: "Owners not frequented by WU generally DO NOT make modifications" that would be completely in line with the conditionals in the stimulus. As is, however, (A) inserts RELUCTANCE which is an idea that cannot be assumed. Why? Because, I believe it makes MORE sense, that if owners are not frequented by WU, they would be LESS reluctant and more INTERESTED in installations of ramps because the stimulus clearly states that with installations of ramps, comes more WU. So (A) is in fact, the opposite/contradicted.
User avatar
 
rinagoldfield
Thanks Received: 308
Atticus Finch
Atticus Finch
 
Posts: 390
Joined: December 13th, 2011
 
 
 

Re: Q13 - Editorialist : Some people argue that ramps and other

by rinagoldfield Wed Jul 29, 2015 12:28 pm

Hi Robinzhang7,

Thanks for your post. Note that the editorialist’s statement as you diagrammed it is NOT the contrapositive of the view held by “some people.” “Installations made” is not the negation of “installations unnecessary.” A true contrapositive of the view expressed by some people is:

Installations necessary --> frequented by WU

This is not what the stimulus says, which is:

Installations made --> frequented

(A) is incorrect for two reasons. Firstly, it talks about what business owners feel. The stimulus never identifies a business-owner point of view. Secondly, as you note, (A) talks about “reluctance” while the stimulus talks about the necessity of certain changes. This is a subtle but crucial difference.
 
jm.kahn
Thanks Received: 10
Elle Woods
Elle Woods
 
Posts: 88
Joined: September 02nd, 2013
 
 
 

Re: Q13 - Editorialist : Some people argue that ramps and other

by jm.kahn Tue Sep 15, 2015 10:37 pm

Why is the credited answer choice E assuming causation when the stimulus only claims correlation? Isn't that a big logical leap to make?

The stim only says "once accommodations installed, wheelchair people start coming", which doesn't seem to mean causation. Any expert help?